If there is one ranch house directly opposite another ranch house, which one of the following could be true?

yckim2180 on July 16, 2021

Please help

How do we arrive at answer B?

Replies
Create a free account to read and take part in forum discussions.

Already have an account? log in

Emil-Kunkin on December 1 at 03:10PM

From our initial setup, we can tell that 5 must be a t, since it cannot be r or s, and that 8 must be a t, because it can't be another s, and putting an r there would violate the r/t rule. So our initial setup is

_ R T _
_ _ S T

However here, we are given the condition that there are two rs facing each other. The only two that could fit this bill are. 3 and 5, so we have

_ R T _
_ R S T

We also know that since we must have a T next to every R, spot 2 must be a T. This eliminates every option but b

Emil-Kunkin on December 1 at 03:11PM

Sorry, spot 2 must be T, which eliminates everything but b.m